How to Use Stokes' Theorem for Evaluating Line Integrals?

Click For Summary
Stokes' Theorem is applied to evaluate the line integral of the given vector field along the curve defined by the intersection of a paraboloid and a plane. The user initially struggled with the setup but identified a mistake in calculating the surface element, correcting it to dS = (1)dA. After fixing the error, the user arrived at the final answer of 135π/2. Additional advice was provided to check the z component of the curl of the vector field for simplification. The discussion emphasizes the importance of accurately determining the surface element in applying Stokes' Theorem.
jegues
Messages
1,085
Reaction score
3

Homework Statement



Use Stoke's theorem to evaluate the line integral

\oint y^{3}zdx - x^{3}zdy + 4dz

where C is the curve of intersection of the paraboloid z = 2 + x^{2} + y^{2} and the plane z=5, directed clockwise as viewed from the point (0,0,7).

Homework Equations





The Attempt at a Solution



Am I doing everything up to this integral correctly? I'm stuck at where I'm at now.

Did I make a mistake along the way?

EDIT: I found one mistake, since my surface S is Z=5, my dS should simply be (1)dA. I fixed my mistakes and found the answer to be \frac{135\pi}{2}
 

Attachments

  • Stokes.jpg
    Stokes.jpg
    30 KB · Views: 517
Last edited:
Physics news on Phys.org
jegues said:

Homework Statement



Use Stoke's theorem to evaluate the line integral

\oint y^{3}zdx - x^{3}zdy + 4dz

where C is the curve of intersection of the paraboloid z = 2 + x^{2} + y^{2} and the plane z=5, directed clockwise as viewed from the point (0,0,7).

Homework Equations





The Attempt at a Solution



Am I doing everything up to this integral correctly? I'm stuck at where I'm at now.

Did I make a mistake along the way?

EDIT: I found one mistake, since my surface S is Z=5, my dS should simply be (1)dA. I fixed my mistakes and found the answer to be \frac{135\pi}{2}

Check your z component of curl F. You should be able to factor out a z. And with n=-k the integral should come out very easy.
 
Question: A clock's minute hand has length 4 and its hour hand has length 3. What is the distance between the tips at the moment when it is increasing most rapidly?(Putnam Exam Question) Answer: Making assumption that both the hands moves at constant angular velocities, the answer is ## \sqrt{7} .## But don't you think this assumption is somewhat doubtful and wrong?

Similar threads

  • · Replies 7 ·
Replies
7
Views
2K
  • · Replies 2 ·
Replies
2
Views
2K
  • · Replies 2 ·
Replies
2
Views
2K
  • · Replies 1 ·
Replies
1
Views
1K
  • · Replies 6 ·
Replies
6
Views
2K
  • · Replies 4 ·
Replies
4
Views
2K
  • · Replies 8 ·
Replies
8
Views
2K
  • · Replies 3 ·
Replies
3
Views
2K
  • · Replies 21 ·
Replies
21
Views
4K
  • · Replies 4 ·
Replies
4
Views
2K